Difference between revisions of "2021 Fall AMC 10A Problems/Problem 10"

m
(Tag: New redirect)
 
Line 1: Line 1:
==Problem==
+
#REDIRECT [[2021_Fall_AMC_12A_Problems/Problem_7]]
A school has <math>100</math> students and <math>5</math> teachers. In the first period, each student is taking one class, and each teacher is teaching one class. The enrollments in the classes are <math>50, 20, 20, 5, </math> and <math>5</math>. Let <math>t</math> be the average value obtained if a teacher is picked at random and the number of students in their class is noted. Let <math>s</math> be the average value obtained if a student was picked at random and the number of students in their class, including the student, is noted. What is <math>t-s</math>?
 
 
 
<math>\textbf{(A)}\ {-}18.5  \qquad\textbf{(B)}\  {-}13.5 \qquad\textbf{(C)}\  0 \qquad\textbf{(D)}\
 
13.5 \qquad\textbf{(E)}\ 18.5</math>
 
 
 
==Solution==
 
The formula for expected values is <cmath>\text{Expected Value}=\sum(\text{Outcome}\cdot\text{Probability}).</cmath>
 
We have
 
<cmath>\begin{align*}
 
t &= \frac15\cdot50 + \frac15\cdot20 + \frac15\cdot20 + \frac15\cdot5 + \frac15\cdot5 \\
 
&= \frac15\cdot(50+20+20+5+5) \\
 
&= \frac15\cdot100 \\
 
&= 20, \\
 
s &= \frac{50}{100}\cdot50 + \frac{20}{100}\cdot20 + \frac{20}{100}\cdot20 + \frac{5}{100}\cdot5 + \frac{5}{100}\cdot5 \\
 
&= 25 + 4 + 4 + 0.25 + 0.25 \\
 
&= 33.5.
 
\end{align*}</cmath>
 
Therefore, the answer is <math>t-s=\boxed{\textbf{(B)}\  {-}13.5}.</math>
 
 
 
~MRENTHUSIASM
 
 
 
==See Also==
 
{{AMC10 box|year=2021 Fall|ab=A|num-b=9|num-a=11}}
 
{{MAA Notice}}
 

Latest revision as of 20:07, 23 November 2021